PT50.S2.Q21- illness caused by bacteria in seafood

JSJS Free Trial Member
edited December 2015 in Logical Reasoning 184 karma
For this question here:
http://7sage.com/lsat_explanations/lsat-50-section-2-question-21/

My question:
If E is right, it didn't resolve this discrepancy. Because E implies "if all and only those who ate a particular seafood dish at the restaurant contracted the illness", which only prove that contaminated dish caused the illness. But what official believe is that "contaminated seafood caused the cases of illness". "seafood" and "dish" is not a same thing.

Comments

  • paulfan2011paulfan2011 Free Trial Member
    125 karma
    Well, it is the same thing. The dish is made from seafood.
  • JSJS Free Trial Member
    184 karma
    @paulfan2011, if it is the same thing, why in the stimulus we know careful testing showed that most people who ate seafood at the restaurant on that date had not come in contract with the bacteria, which seems contradict with E "all and only those who ate a particular seafood dish at the restaurant contracted the illness. "
  • paulfan2011paulfan2011 Free Trial Member
    125 karma
    Because they did not eat the dish that has the disease. Say that the restaurant serves several seafood dishes and one of the dishes has the disease. E says only people who eat the contaminated dish got sick.

    The stimulus says most people eat seafood at the restaurant, this does not mean most people ate the contaminated dish
  • JSJS Free Trial Member
    184 karma
    So it means that those who eat a special seafood dishes only occupy a small share of the whole people who eat seafood in that restaurant ? which I think explain why Careful testing showed that most people who ate seafood at the restaurant on that date had not come in contact with the bacteria in question?
  • paulfan2011paulfan2011 Free Trial Member
    125 karma
    Yea.
  • JSJS Free Trial Member
    184 karma
    wow, @paulfan2011, I found you become peer instructor! Congrats! I envy your title! :)
  • KK Free Trial Member
    edited August 2013 345 karma
    grats paulfan

    hey not to hijack this thread but since we're talking about a question from PT 50 can u help me with S4 #13?

    I am unable to connect the premises to the conclusion the way JY taught.

    I picked B after eliminating everything else. Is that correct?
  • JSJS Free Trial Member
    184 karma
    @K, haha, now I can be a peer instructor.
    Take easy, this question is quite simple cause it is sufficient assumption question and we can totally predict that.

    In the question stimulus, we know journalist recycled the old photographs, which seems to overstates the similarities between past and present and they had been accused of distorting public understanding.

    How can we know journalist distorting public understanding just because repetition of old picture? So we need a bridge here: if overstating the similarities-->distorting public understanding. Just search right answer, it's B, Bingo!!!

    Hope helps :)
  • KK Free Trial Member
    345 karma
    Thanks! Ugh I have another question if that's okay.

    #18 from section 4.

    Out of the seven questions I got wrong in the LR from this test, this one was the hardest to grasp.

    I thought A was irrelevant.
  • JSJS Free Trial Member
    184 karma
    You mean the TSX-400 pesticide one?

    This question is quite tough. I think JY definitely should post video for this question.

    Also, pseudo-sufficent question we can definitely predict the answer.

    In the stimulus, we know that one legal pesticide is more harmful than illegal pesticide, then either both pesticide should be banned or both should be legalized.

    For D, if we do the contra-positive, we can get:

    legal one is more harmful than illegal one or legal one is as harmful as illegal one, then both should be banned or both legalized.

    That's exactly what the conclusion says.

    Hope helps:)
  • KK Free Trial Member
    345 karma
    Thanks that was really very helpful.

    I had the stimulus diagrammed but I had trouble with elimination. I got rid of A and E but was left with C,B,D
  • JSJS Free Trial Member
    184 karma
    @K, B and C is very easy to eliminate cuz from stimulus we know we should look for the conclusion like XXX--->both banned or both legal.(necessary condition). But in B and C, the y both put this one in the sufficient condition, which is definitely wrong. You can eliminate these two choices immediately.

    Hope this method helps :)
  • KK Free Trial Member
    345 karma
    Ohhhhh my god... wow...

    I can't believe I didn't understand that from that answer choice...

    Thanks a lot JS :D
  • JSJS Free Trial Member
    184 karma
    haha, @K, good luck !
  • paulfan2011paulfan2011 Free Trial Member
    125 karma
    Ah, I remember JS and I discussing this question.
    Here is my understanding of this question, reproduced from my email to JS:

    Sufficient condition: one pesticide should be legal and another illegal.

    Necessary condition: the legal pesticide is less harmful to the environment than the illegal pesticide.

    Premise: the legal pesticide is NOT less harmful to the environment than the illegal pesticide. (it's actually more harmful)

    Necessary condition fails, negate sufficient.

    Conclusion: it is not the case that one pesticide should be legal and another illegal.

    This is logically equivalent to the conclusion of the stimulus.

    Given two products T (TSX-400) and EZ (Envirochem and Zanar) either one could be legal or illegal, but not both.

    There are four possible arrangements:
    1.)T legal EZ legal
    2.)T legal EZ illegal
    3.)T illegal EZ legal
    4.)T illegal EZ illegal

    It is not the case that one pesticide should be legal and another illegal eliminates 2 and 3, leaving 1 and 4.

    Now to the actual conclusion. (If these studies are accurate, then either Envirochem and Zanar should be banned or TSX-400 should be legalized) Currently, T is illegal and EZ is legal, and the conclusion suggests two mutually exclusive solutions.

    1.) We ban EZ. T illegal EZ illegal (arrangement 4)
    OR
    2.) We legalize T. T legal EZ legal (arrangement 1)

    You can see that answer choice D and the actual conclusion is logically equivalent.
  • KK Free Trial Member
    345 karma
    Thanks for the information Paul
  • J.Y. PingJ.Y. Ping Administrator Instructor
    14050 karma
    I like paulfan2011's explanation. Good job.

    For those who have access, here's the video I did:
    http://7sage.com/lsat_explanations/lsat-50-section-4-question-18

    Very tough question.
Sign In or Register to comment.